Expressing an Arbitrary Vector in Terms of Noncoplanar Vectors

Click For Summary
An arbitrary vector V can be expressed in terms of three noncoplanar vectors A, B, and C using the formula V = [V,B,C]A/[A,B,C] + [V,C,A]B/[A,B,C] + [V,A,B]C/[A,B,C]. The hint suggests expressing V as a linear combination of A, B, and C, which involves finding the coefficients a, b, and c through scalar products. The discussion clarifies that the notation [V,B,C] represents the scalar triple product, which simplifies the calculations. By recognizing that the components of the dot product with B and C vanish, the solution can be derived effectively. The hint ultimately guides the user to the correct approach for solving the problem.
EngageEngage
Messages
203
Reaction score
0

Homework Statement


Show that an arbitrary vector V can be expressed in terms of any three noncoplanar vectors, A, B, C, according to:

V = [V,B,C]A/[A,B,C] + [V,C,A]B/[A,B,C] + [V,A,B]C/[A,B,C]


Homework Equations


A Hint is given:
We know that V can be expressed as aA + bB +cC; to find a, take the scalar product of V with BxC


The Attempt at a Solution


I tried to solve this one by relating the projections of V to 3 arbitrary vectors, A, B, C, but I couldn't get to the answer above. I'm also not sure how the hint will help me either. Could someone please help me get started on this because I am all out of ideas.

thanks
 
Physics news on Phys.org
I'm assuming [V,B,C] is the triple product V.(BxC), right? Then what is [V,B,C]? It's (aA+bB+cC).(BxC). BxC is perpendicular to B and C, so those parts of the dot product are zero. This leaves you with [V,B,C]=aA.(BxC)=a*[A,B,C]. Put that into your formula and treat the other two terms the same way.
 
Wow, so the hint gave it away -- i can't believe i didn't see that. Thanks a lot for the help though!
 
Question: A clock's minute hand has length 4 and its hour hand has length 3. What is the distance between the tips at the moment when it is increasing most rapidly?(Putnam Exam Question) Answer: Making assumption that both the hands moves at constant angular velocities, the answer is ## \sqrt{7} .## But don't you think this assumption is somewhat doubtful and wrong?

Similar threads

  • · Replies 5 ·
Replies
5
Views
2K
  • · Replies 6 ·
Replies
6
Views
3K
  • · Replies 4 ·
Replies
4
Views
2K
  • · Replies 2 ·
Replies
2
Views
2K
  • · Replies 2 ·
Replies
2
Views
2K
  • · Replies 8 ·
Replies
8
Views
2K
Replies
1
Views
1K
Replies
6
Views
8K
  • · Replies 16 ·
Replies
16
Views
6K
  • · Replies 2 ·
Replies
2
Views
3K